don't get any of this ​

Don't Get Any Of This

Answers

Answer 1

Answer:

the value of the printer is going down over the years because it is getting older. all you need to do for this is label the x and y axis with the nbers on the graph and then fill it in

has ur teacher gone over how to fill in graphs yet?

Answer 2
part a- represent V as a function of t

step 1: find the slope by picking any two coordinates and using the slope formula
m=(y2-y1)/(x2-x1)

-i chose (8,2000) and (4,7000) but it doesn't matter what you choose.

-plug in:
m=(7000-2000)/(4-8)=-5000/4=-1250

step 2: plug any coordinate into equation y=mx+b

- i chose (0,12000)

- plug in:
(12000)=(-1250)(0)+b

- solve for b
b=12000

step 3: V as a function of t
- use slope intercept equation y=mx+b
- V(t)=y
- t=x
- m=-1250
- b=12000
V(t)=-1250t+12000 (final answer)

part b- graph it

part c- the slope represents the rate at which the copier decreases in value over t years.


Related Questions

The tables show information about the scale ratios of two different maps. 2 tables. The first table is a 2-column table with 2 rows titled Map of the United States. Column 1 is labeled Centimeters with entries 3, 12. Column 2 is labeled Miles with entries 50, 200. The second table is a 2-column table with 2 rows titled Map of Africa. Column 1 is labeled Centimeters with entries 2, 15. Column 2 is labeled miles with entries 40, 300. The ratio of centimeters to miles on the map of the United States is . The ratio of centimeters to miles on the map of Africa is . The ratio of centimeters to miles on the map of the United States is the ratio of centimeters to miles on the map of Africa.

Answers

The answers are in order 3/50,1/20, greater than.

Answer:

1.b 2.a 3.b

Step-by-step explanation:

help pls. Find the value of x.

Answers

Answer:

[tex] \boxed{ \bold{ \huge{ \boxed{ \sf{x = 15 °}}}}}[/tex]

Step-by-step explanation:

[tex] \sf{x + 30 ° + 100 ° + 3x - 10 ° = 180 °}[/tex]

( Sum of angle in straight line )

Collect like terms

⇒[tex] \sf{4x + 30 ° + 100 ° - 10 ° = 180 °}[/tex]

Calculate the sum or difference

⇒[tex] \sf{4x + 120 ° = 180 °} [/tex]

Move 120 to right hand side hand change it's sign

⇒[tex] \sf{4x = 180 ° - 120 °}[/tex]

Subtract 120 from 180

⇒[tex] \sf{4x = 60 ° }[/tex]

Divide both sides of the equation by 4

⇒[tex] \sf{ \frac{4x}{4} = \frac{60}{4} }[/tex]

Calculate

⇒[tex] \sf{x = 15  °}[/tex]

Hope I helped!

Best regards!!

Answer: x = 15 degree

Explanation:

(x + 30) + 100 + (3x - 10) = 180 (straight angle)

x + 30 + 100 + 3x - 10 = 180
4x + 120 = 180
4x = 180 - 120
4x = 60
x = 60/4
x = 15 degree

Therefore, x = 15 degree

please help asap

solve x^4+5x^2+4=0

Answers

Answer:

[tex]x=\pm i\text{ or }x=\pm2i[/tex]

Step-by-step explanation:

So we have the equation:

[tex]x^4+5x^2+4=0[/tex]

Let's let u be equal to x². So:

[tex]u^2+5u+4=0[/tex]

Factor:

[tex](u+1)(u+4)=0[/tex]

Zero Product Property:

[tex]u+1=0\text{ or } u+4=0[/tex]

Subtract:

[tex]u=-1\text{ or }u=-4[/tex]

Replace:

[tex]x^2=-1\text{ or }x^2=-4[/tex]

Take square root:

[tex]x=\pm\sqrt{-1}\text{ or }x=\pm\sqrt{-4[/tex]

Simplify:

[tex]x=\pm i\text{ or }x=\pm2i[/tex]

What part of the expression is a factor? 12+5(-3n-8)

Answers

Answer:

5

Step-by-step explanation:

A factor is a number or an expression that is multiplying something.

5 is multiplying -3n + 8.

Answer: 5 is a factor.

Pls help!! I need the answer immediately

Answers

Answer:

i). x³ + 9x² + yz - 15

ii). -21m³np - 8p⁵q + mnp + 4mn + 100

Step-by-step explanation:

Question (38)

i). Two expressions are -5x² - 4yz + 15 and x³+ 4x²- 3yz

  By subtracting expression (1) from expression (2) we can the expression by addition which we can get expression (1).

 (x³+ 4x²- 3yz) - (-5x² - 4yz + 15) = x³ + 4x² - 3yz + 5x² + 4yz - 15

                                                    = x³ + 9x² + yz - 15

ii). -15m³np + 2p⁵q - 6m³pn + mnp + 4mn - 10qp⁵+ 100

  = (-15m³np - 6m³np) + (2p⁵q - 10qp⁵) + mnp + 4mn + 100

  = -21m³np - 8p⁵q + mnp + 4mn + 100

How to Multiply six and the reciprocal of the quotient of a number and seven?

Answers

Answer:

Let N be the unknown number

The we have 6 x reciprocal of N/7  which is 6 x 7/N which is 42/N

HOPE THIS HELPS YOU

Multiply six and the reciprocal of the quotient of a number and seven can be written as 42/M.

Where M is the number.

Given,

Multiply six and the reciprocal of the quotient of a number and seven.

We need to find this expression.

What does quotient mean?

It means to divide.

The quotient of a and b:

=  a ÷ b.

= a / b

Find the expression.

Let the number be M.

Now,

The quotient of a number and 7 can be written as:

= M ÷ 7

= M / 7

Reciprocal of the quotient of a number and seven.

= 1 / (M/7)

= 7 / M

Now,

Multiply six and the reciprocal of the quotient of a number and seven.

It can be written as:

= 6 x 7 / M

= 42 / M

Thus multiply six and the reciprocal of the quotient of a number and seven can be written as 42/M.

Learn more about the quotient of a number and its reciprocal here:

https://brainly.com/question/673545

#SPJ2

Imagine that you earned $8,425 in one year. If the government enforces a 15% income tax, how much money would you owe in taxes at the end of the year?

Answers

Answer: You would owe $1,263 and 75c! Have a fantastic day!

Solve for x. 2x + 2 = -8 X=???? Please help!?

Answers

Answer:

x = - 5

Step-by-step explanation:

2x + 2 = - 8

2x = - 8 - 2

2x = - 10

x = - 10 : 2

x = - 5

Answer:

x=-5

Step-by-step explanation:

1.) 2x+2=-8   start to isolate the variable by subtracting 2 to -8

2.) 2x=-10 divide both sides by 2

3.) x=-5

Hope this helped

Complete the sequence,
13, 14, 27, 41,

Answers

Answer:

68

Step-by-step explanation:

We can see that the nth term would be the sum of the previous two terms. Continuing this would result in 68.

Four more than twice a number is –10.

Answers

Four more than twice a number is –10 is the same as 2n + 4 = 10

Steps to solve:

2n + 4 = 10

~Subtract 4 to both sides

2n = 6

~Divide 2 to both sides

n = 3

Best of Luck!

Aieeeee I was looking for an answer to this

Which function has a domain of x 25 and a range of y s3?

Answers

Answer:

Step-by-step explanation:

The function  has a domain x ≥ 5.

This is because the function remains real for (x - 5) ≥ 0 as negative within the square root is imaginary.

Hence, (x - 5) ≥ 0

⇒ x ≥ 5

Now, for all x values that are greater than equal to 5 the value of  will be negative.

So,  

⇒  

⇒ y ≤ 3

Therefore, the range of the function is y ≤ 3. (Answer)

Note: Thanks rani

i need help with the top one

Answers

Answer:

2. 14

4. -6

Step-by-step explanation:

#2

(x²-4y)/2, given x=4, y=-3

 [(4)²-4(-3)]/2

=[16+12]/2

=28/2

=14

------------------------

#4

19-x², given x=-5

 19-(-5)²

=19-25

=-6

Find the distance between two numbers on the number line -3 1/4, -6 1/2

Answers

Answer:

[tex]3\frac{1}{4}[/tex]

Step-by-step explanation:

Given the two  -3 1/4 and -6 1/2 numbers  on the line, the distance between he numbers can be gotten by simply taking the differences between both values as shown;

[tex]= -3\frac{1}{4} -(-6\frac{1}{2} )\\ \\= -\frac{13}{4} + \frac{13}{2}\\ \\= \frac{-13+26}{4}\\ \\= \frac{13}{4} \\\\= 3\frac{1}{4}[/tex]

Hence the distance between two numbers on the number line is [tex]3\frac{1}{4}[/tex]

Hours of
Training
10
20
30
40
50
60
70
Monthly
Pay
1220
1420
1620
1820
2020
2220
2420
According to the table, what is the monthly pay for a worker with 0 hours of
training?

Answers

Answer:

$1,020

Step-by-step explanation:

There is a $200 difference in the every monthly pay.

Because of this we know that the first month was an amount minus $200.

If we subtract 1220 and 200, you get the pay of 1020 for 0 hours.

£1020

There is a £200 difference between every hour and pay

Find the cosine ratio of angle 0

Answers

Answer: cos(0) = 1

hope this helped

Can somebody help me with this ?? :)

Answers

only the first table represents a function because no repeating x’s can have different y values

Exercises 5-8 refer to a bag filled with 3 yellow balls, 4 green balls, 2 blue balls, and 1 orange ball.

S.What is the probability of picking a red ball out of the bag?

6. What is the probability of picking a blue or yellow ball?

7. What is the probability of picking an orange ball?

8.What is the probability of picking a green ball?

9. What is the probability of picking a green or orange ball?

Answers

Answer:

s. 0/10

6. 5/0

7. 1/10

8. 4/10

9. 5/10

find the mean of first five prime
numbers​

Answers

Answer:

5.6

Step-by-step explanation:

(2+3+5+7+11)/5

=5.6

Given: 2x + 3(x - 2) = 24
Prove: x = 6
Statements and Reasons?

Answers

Answer:

x=6

Step-by-step explanation:

first we'll use distributive property: we will distribute and multiply 3 by x and - 2

2x + 3x -6 = 24

then we'll add like terms which are 2x and 3x , 24 and 6 the sign of 6 will change from negative to positive:

2x + 3x =24 + 6

5x = 30

after that we will divide both sides by 5 in order to get x

5x/5 = 30/ 5

x = 6

Which pair of numbers has 7 as its least common multiple? Group of answer choices 1, 7 3, 4 7, 21 14, 28

Answers

Answer:

7, 14, 14,21, and 28

Step-by-step explanation:

Only 1, 7 is pair of numbers that has 7 as its least common multiple.

Given that,
Pairs ( 1, 7 ), ( 3, 4 ), ( 7, 21), ( 14, 28).
A pair or pairs to be determined that have 7 as its least common multiple.

What is arithmetic?

In mathematics, it deals with numbers of operations according to the statements.

What is Lowest Common Multiple?

In arithmetic, LCM is defined as the lowest common multiple of two integers X and Y, usually represented as by LCM X, Y, and is the smallest favorable integer that is dividable by both X and Y.

Since the Least common multiple of pair of numbers has 7, this is only possible for 1 * 7, because the LCM of 1 and7 is 7. So from given option 1, 7 is the required pair.

Thus, only 1, 7 is pair of numbers that has 7 as its least common multiple.

Learn more about arithmetic here:

brainly.com/question/14753192

#SPJ2


What two rational expressions sum to 2x+3/x^2-5x+4

Enter your answer by filling in the boxes. Enter your answer so that each rational expression is in simplified form.

Answers

Answer:

[tex] \frac{2x + 3}{(x- 1)(x - 4)} = \frac{-5}{3(x- 1)} + \frac{11}{3(x - 4)} [/tex]

Step-by-step explanation:

Given the rational expression: [tex] \frac{2x + 3}{x^2 - 5x + 4} [/tex], to express this in simplified form, we would need to apply the concept of partial fraction.

Step 1: factorise the denominator

[tex] x^2 - 5x + 4 [/tex]

[tex] x^2 - 4x - x + 4 [/tex]

[tex] (x^2 - 4x) - (x + 4) [/tex]

[tex] x(x - 4) - 1(x - 4) [/tex]

[tex] (x- 1)(x - 4) [/tex]

Thus, we now have: [tex] \frac{2x + 3}{(x- 1)(x - 4)} [/tex]

Step 2: Apply the concept of Partial Fraction

Let,

[tex] \frac{2x + 3}{(x- 1)(x - 4)} [/tex] = [tex] \frac{A}{x- 1} + \frac{B}{x - 4} [/tex]

Multiply both sides by (x - 1)(x - 4)

[tex] \frac{2x + 3}{(x- 1)(x - 4)} * (x - 1)(x - 4) [/tex] = [tex] (\frac{A}{x- 1} + \frac{B}{x - 4}) * (x - 1)(x - 4) [/tex]

[tex] 2x + 3 = A(x - 4) + B(x - 1) [/tex]

Step 3:

Substituting x = 4 in [tex] 2x + 3 = A(x - 4) + B(x - 1) [/tex]

[tex] 2(4) + 3 = A(4 - 4) + B(4 - 1) [/tex]

[tex] 8 + 3 = A(0) + B(3) [/tex]

[tex] 11 = 3B [/tex]

[tex] \frac{11}{3} = B [/tex]

[tex] B = \frac{11}{3} [/tex]

Substituting x = 1 in [tex] 2x + 3 = A(x - 4) + B(x - 1) [/tex]

[tex] 2(1) + 3 = A(1 - 4) + B(1 - 1) [/tex]

[tex] 2 + 3 = A(-3) + B(0) [/tex]

[tex] 5 = -3A [/tex]

[tex] \frac{5}{-3} = \frac{-3A}{-3} [/tex]

[tex] A = -\frac{5}{3} [/tex]

Step 4: Plug in the values of A and B into the original equation in step 2

[tex] \frac{2x + 3}{(x- 1)(x - 4)} = \frac{A}{x- 1} + \frac{B}{x - 4} [/tex]

[tex] \frac{2x + 3}{(x- 1)(x - 4)} = \frac{-5}{3(x- 1)} + \frac{11}{3(x - 4)} [/tex]

Answer:

Step-by-step explanation:

Given the rational expression: , to express this in simplified form, we would need to apply the concept of partial fraction.

Step 1: factorise the denominator

Thus, we now have:

Step 2: Apply the concept of Partial Fraction

Let,

=

Multiply both sides by (x - 1)(x - 4)

=

Step 3:

Substituting x = 4 in

Substituting x = 1 in

Step 4: Plug in the values of A and B into the original equation in step 2

Step-by-step explanation:

please someone help me to prove this...

Answers

Step-by-step explanation:

cos 20° cos 40° cos 60° cos 80°

Simplify cos 60° to ½.

½ cos 20° cos 40° cos 80°

We can use double angle formula, sin(2θ) = 2 sin θ cos θ.  Multiply and divide by 2 sin 20°.

½ (2 sin 20° cos 20° cos 40° cos 80°) / (2 sin 20°)

½ (sin 40° cos 40° cos 80°) / (2 sin 20°)

Multiply and divide by 2 and use double angle formula again.

½ (2 sin 40° cos 40° cos 80°) / (4 sin 20°)

½ (sin 80° cos 80°) / (4 sin 20°)

Multiply and divide by 2 and use double angle formula again.

½ (2 sin 80° cos 80°) / (8 sin 20°)

½ (sin 160°) / (8 sin 20°)

Use phase shift identity sin θ = sin(180−θ).

½ (sin 20°) / (8 sin 20°)

1/16

In this case we have the equation cos(20°)(cos(40°)(cos(60°)(cos(80°) = 1 / 16, and are asked to prove that this equation is true. Let's start by using the property 'cos(s)cos(t) = cos(s + t) + cos(s - t) / 2.' Taking the bit 'cos(20°)(cos(40°)' we know that s should = 20°, and t should = 40°.

[tex]\mathrm{Using\:the\:following\:identity}:\quad \cos \left(s\right)\cos \left(t\right)=\frac{\cos \left(s+t\right)+\cos \left(s-t\right)}{2},[/tex]

[tex]\cos \left(20^{\circ \:}\right)\cos \left(40^{\circ \:}\right)=\frac{\cos \left(20^{\circ \:}+40^{\circ \:}\right)+\cos \left(20^{\circ \:}-40^{\circ \:}\right)}{2}\\[/tex]

[tex]\mathrm{Substituting\:the\:value\:back}:\frac{\cos \left(20^{\circ \:}+40^{\circ \:}\right)+\cos \left(20^{\circ \:}-40^{\circ \:}\right)}{2}\cos \left(60^{\circ \:}\right)\cos \left(80^{\circ \:}\right)[/tex]

[tex]\mathrm{Multiply\:fractions}:\quad \frac{\cos \left(60^{\circ \:}\right)\cos \left(80^{\circ \:}\right)\left(\cos \left(60^{\circ \:}\right)+\cos \left(-20^{\circ \:}\right)\right)}{2}[/tex]

This expression is indeed not simplified, but remember that we can use the identities 'cos(- 20°) = cos(20°)' and 'cos(60°) = 1 / 2.' Let's substitute one step and a time.

[tex]\cos \left(-20^{\circ \:}\right)=\cos \left(20^{\circ \:}\right): \frac{\cos \left(60^{\circ \:}\right)\cos \left(80^{\circ \:}\right)\left(\cos \left(60^{\circ \:}\right)+\cos \left(20^{\circ \:}\right)\right)}{2}[/tex]

[tex]\cos \left(60^{\circ \:}\right)=\frac{1}{2}:\frac{\frac{1}{2}\cos \left(80^{\circ \:}\right)\left(\frac{1}{2}+\cos \left(20^{\circ \:}\right)\right)}{2}[/tex]

[tex]\frac{\frac{1}{2}\cos \left(80^{\circ \:}\right)\left(\frac{1}{2}+\cos \left(20^{\circ \:}\right)\right)}{2} = \frac{\cos \left(80^{\circ \:}\right)\left(1+2\cos \left(20^{\circ \:}\right)\right)}{8}[/tex]

Now that we have this 'simplified expression,' if we take the numerator as a whole in decimal form, it will = 0.5. Respectively 0.5 / 8 = 1 / 2 / 8 = 1 / 16. Hence our equation is true.

[tex]\cos \left(80^{\circ \:\:}\right)\left(1+2\cos \left(20^{\circ \:\:}\right)\right) = 0.5,\\0.5 / 8 = (1 / 2) / 8 = 1 / 16[/tex]

Which is the function represented by the table?

Answers

The answer is A, you would plug in 3 for fx so therefore the answer would be A. Then to make sure plug in the other functions.:)

Answer:

f(x)= -2x + 1

Step-by-step explanation:

The function we will find is to be written in slope-intercept form, which is y=mx+b.

One way to find the answer would be by graphing the function, wich you can do on an online calculator or a sheet of paper.

Another way to get to our answer without even having to graph is by finding the y-intercept, or the b, of the function given in the table.

We can do this by looking at the 0 value of the table. The table tells us that the y-intercept is 1.

So, we have to look at our options and see which one has +1 for the y-intercept.

The option f(x)=-2x+1 is the answer

How to check your work: input the given x values into the chosen function, in this case f(x)= -2x + 1

So for example, you could check that this answer is the function by doing

f(-1) = -2(-1) + 1f(-1) = 2 + 1 f(-1) = 3

Make sure to do this to all values.

Hope this helps!

∠A and \angle B∠B are complementary angles. If m\angle A=(2x+10)^{\circ}∠A=(2x+10) ∘ and m\angle B=(3x+15)^{\circ}∠B=(3x+15) ∘ , then find the measure of \angle B∠B.

Answers

Answer:

[tex]54^\circ[/tex]

Step-by-step explanation:

Given :

[tex]m\angle A=(2x+10)^{\circ}[/tex]

[tex]m\angle B=(3x+15)^{\circ}[/tex]

And angles [tex]\angle A[/tex] and [tex]\angle B[/tex] are complementary angles.

To find:

The measure of [tex]\angle B[/tex].

Solution:

First of all, let us learn about complementary angles.

Complementary angles are the angles whose sum is equal to [tex]90^\circ[/tex].

And we are given that angles [tex]\angle A[/tex] and [tex]\angle B[/tex] are complementary angles.

Therefore [tex]\angle A[/tex] + [tex]\angle B[/tex] = [tex]90^\circ[/tex]

[tex]\Rightarrow 2x+10+3x+15=90\\\Rightarrow 5x+25=90\\\Rightarrow 5x=65\\\Rightarrow x =13[/tex]

Putting the value in [tex]\angle B[/tex].

[tex]m\angle B=(3\times 13+15)^{\circ} = 54^\circ[/tex]

Therefore, the answer is [tex]\bold{\angle B = 54^\circ}[/tex]

ray BD bisects ∠ABC m∠ABD if m∠ABC = 5/2 y and m∠DBC = (y+5)degrees

Answers

Step-by-step explanation:

BD bisects ∠ABC.  Therefore:

m∠DBC = ½ m∠ABC

y + 5 = ½ (5/2 y)

y + 5 = 5/4 y

4y + 20 = 5y

y = 20

Therefore, m∠ABC = 50°, and m∠ABD = 25°.

It takes Mr. Martin 12 minutes to drive his bus route without stopping to pick up any passengers. For each passenger stop, he estimates that 30 seconds is added to his travel time. On Tuesday, Mr. Martin spends 14 minutes driving his route.

Answers

The equal it some help it by ethan and martin in 17 kilometer car

solve for a 36a =9 please help asap

Answers

Answer:

a = 1/4

Step-by-step explanation:

Divide each side by 36:

36a = 9

a = 1/4

Answer:

a=1/4=0.25

Step-by-step explanation:

to get the value of a divide 36 by 9.

[tex]36a=9\\ \frac{36a}{36}=\frac{9}{36}\\ a=\frac{9}{36}\\ (simplify) \\ a=\frac{1}{4}=0.25[/tex]

please help i need this answered as soon as possible!!! -Use ΔDEF, shown below, to answer the question that follows:

Answers

Answer:

X= 139

Step-by-step explanation:

the square on the E means its a right side so that would be 90 degrees.  All triangles add up to 180.  F would be 41 degrees.  that mean X is  139 i think.

Answer:

36.083

Step-by-step explanation:

SOHCAHTOA

Cosine is adjacent over hypotenuse, so the

cos(49) = the adjacent side (x) || over the hypotenuse (55)

cos(49) = x/55

55cos(49) = x

x = 36.083

How many inches is 2 1/2 miles

Answers

Answer:

2.5 mile inch 158400

Step-by-step explanation:

It's on google

158400

You can just ask google

Which of the following is closest to the square root of 4000 A) 20 B) 60 C) 400 D) 2000

Answers

Answer:A

Step-by-step explanation:

Sqrt(4000)= 20

Other Questions
What is the lowest area below sea level? what are the factors of 2b (3 c)? What number is 10^2 times as long as 0.012? (4.2 x 103) x (1.3 x 10-1) Twelve is no less Thant he sum of a number n and 3 Research revealed that when drinking alcohol reduced dissonance-produced arousal, the attitudes-follows-behavior effect:___________.A. increased greatly. B. disappeared.C. was unaffected.D. increased slighty. A rifle bullet is fired from the top of a cliff at an angle of 30o below the horizontal. The initial velocity of the bullet is 800 m/s. If the cliff is 80 m high, how far does it travel horizontally Need help quick simplify the expression (-4) power of 3 Look at the graph. On a coordinate plane, a line with negative slope goes through points (0, 2) and (4, 0). To find the rate of change of the function, Kevin did the following: StartFraction 0 minus (negative 1) Over 2 minus 2.5 EndFraction = StartFraction 1 Over negative 0.5 EndFraction = negative 2. What was Kevins mistake? He incorrectly chose (1, 2.5) as a point on the graph. He incorrectly chose (0, 2) as a point on the graph. He mixed up the numerator and the denominator of the fraction. He subtracted 1 from 0 when he should have added 1 to 0. The real number 7/12belongs to which set of numbers?1.irrational numbers2.natural numbers3.integers4.rational numbers What is the answer to a10/a3 ? please help, thank you :) What determine what people look like1. Carbonhydrates2. Lipids3. Nucleic acids4. Proteins Which diverse group of settlers helped the colony achieve the economical goal of mercantilism, sending raw materials back to Great Britain. SalzburgersJewsHighland scots What are the parts of the water system? Label the water wheel to show the matter and forms energy that flow through the system. What is the electron emission produced when light is shone on a substance A firm's strategic choices and leadership approaches need to: a. be rigid and concrete to be effective. b. adapt and change to retain their Based on the three-cell model of global circulation, surface winds "diverge" in the vicinity of: Group of answer choices The answer step by step In which area did the Seminole and Choctaw peoples live in approximately 1500? the Southeast the Plains the Northeast the Great Basin